All Exams  >   Bank Exams  >   IBPS PO Prelims & Mains Preparation  >   All Questions

All questions of Data Sufficiency for Bank Exams Exam

Directions to Solve: In each of the questions below consists of a question and two statements numbered I and II given below it. You have to decide whether the data provided in the statements are sufficient to answer the question. Read both the statements and
Give answer:
- (A) If the data in statement I alone are sufficient to answer the question, while the data in statement II alone are not sufficient to answer the question
- (B) If the data in statement II alone are sufficient to answer the question, while the data in statement I alone are not sufficient to answer the question
- (C) If the data either in statement I alone or in statement II alone are sufficient to answer the question
- (D) If the data given in both statements I and II together are not sufficient to answer the question and
- (E) If the data in both statements I and II together are necessary to answer the question.
Question: How is Tanya(girl) related to the man in the photograph ?
Statements:
I.
Man in the photograph is the only son of Tanya's grandfather.

II.The man in the photograph has no brothers or sisters and his father is Tanya's grandfather.
  • a)
    I alone is sufficient while II alone is not sufficient
  • b)
    Both I and II are sufficient
  • c)
    II alone is sufficient while I alone is not sufficient
  • d)
    Neither I nor II is sufficient
  • e)
    Either I or II is sufficient 
Correct answer is option 'B'. Can you explain this answer?

Rahul Gupta answered
From statement I, we know that the man in the photograph is the only son of Tanya's grandfather. This means that Tanya is either the man's daughter or granddaughter.

From statement II, we know that the man in the photograph has no brothers or sisters and his father is Tanya's grandfather. This confirms that Tanya is the man's daughter or granddaughter.

Combining both statements, we can conclude that Tanya is either the daughter or granddaughter of the man in the photograph.
1 Crore+ students have signed up on EduRev. Have you? Download the App

Directions to Solve: In each of the questions below consists of a question and two statements numbered I and II given below it. You have to decide whether the data provided in the statements are sufficient to answer the question. Read both the statements and
Give answer:
- (A) If the data in statement I alone are sufficient to answer the question, while the data in statement II alone are not sufficient to answer the question
- (B) If the data in statement II alone are sufficient to answer the question, while the data in statement I alone are not sufficient to answer the question
- (C) If the data either in statement I alone or in statement II alone are sufficient to answer the question
- (D) If the data given in both statements I and II together are not sufficient to answer the question and
- (E) If the data in both statements I and II together are necessary to answer the question.
Question: How many children does M have ?
Statements:
I.
H is the only daughter of X who is wife of M.

II.K and J are brothers of M.
  • a)
    II alone is sufficient while I alone is not sufficient
  • b)
    Neither I nor II is sufficient
  • c)
    Either I or II is sufficient
  • d)
    I alone is sufficient while II alone is not sufficient
  • e)
    Both I and II are sufficient
Correct answer is option 'D'. Can you explain this answer?

Ishita Reddy answered
The correct answer is:
d) I alone is sufficient while II alone is not sufficient
Explanation:
  • Statement I: H is the only daughter of X, who is the wife of M. This means M has at least one child (H, the daughter). Therefore, this statement alone is sufficient to answer that M has one child.
  • Statement II: K and J are brothers of M. This information provides no direct insight into how many children M has, as it talks about M's brothers, not M's children. Therefore, this statement alone is not sufficient to answer the question.
Hence, Statement I alone is sufficient, while Statement II alone is not sufficient to answer the question.

Directions to Solve: In each of the questions below consists of a question and two statements numbered I and II given below it. You have to decide whether the data provided in the statements are sufficient to answer the question. Read both the statements and
Give answer:
- (A) If the data in statement I alone are sufficient to answer the question, while the data in statement II alone are not sufficient to answer the question
- (B) If the data in statement II alone are sufficient to answer the question, while the data in statement I alone are not sufficient to answer the question
- (C) If the data either in statement I alone or in statement II alone are sufficient to answer the question
- (D) If the data given in both statements I and II together are not sufficient to answer the question and
- (E) If the data in both statements I and II together are necessary to answer the question.
Question: In which year was Rahul born ?
Statements: I.
Rahul at present is 25 years younger to his mother.

II.Rahul's brother, who was born in 1964, is 35 years younger to his mother.
  • a)
    I alone is sufficient while II alone is not sufficient
  • b)
    II alone is sufficient while I alone is not sufficient
  • c)
    Either I or II is sufficient
  • d)
    Neither I nor II is sufficient
  • e)
    Both I and II are sufficient
Correct answer is option 'E'. Can you explain this answer?

From both I and II, we find that Rahul is (35 - 25) = 10 years older than his brother, who was born in 1964. So, Rahul was born in 1954.

Directions to SolveIn each of the questions below consists of a question and two statements numbered I and II given below it. You have to decide whether the data provided in the statements are sufficient to answer the question. Read both the statements andGive answer
 (A) If the data in statement I alone are sufficient to answer the question, while the data in statement II alone are not sufficient to answer the question
(B) If the data in statement II alone are sufficient to answer the question, while the data in statement I alone are not sufficient to answer the question
 (C) If the data either in statement I alone or in statement II alone are sufficient to answer the question
(D) If the data given in both statements I and II together are not sufficient to answer the question and (E) If the data in both statements I and II together are necessary to answer the question.
Question - 
Question: What does '$' mean in a code language?
Statements:I.'5$#3' means 'flowers are really good'.II.'7#35' means 'good flowers are available'.
a)I alone is sufficient while II alone is not sufficient
b)II alone is sufficient while I alone is not sufficient
c)Either I or II is sufficient
d)Neither I nor II is sufficient
e)Both I and II are sufficient
Correct answer is option 'E'. Can you explain this answer?

Jaya Gupta answered
In I and II, the common codes are '5', '#' and '3' and the common words are 'flowers', 'are' and 'good'. Thus, in I, the remaining code '$' stands for 'really'.

Directions to Solve
In each of the questions below consists of a question and two statements numbered I and II given below it. You have to decide whether the data provided in the statements are sufficient to answer the question. Read both the statements and
Give answer
  • (A) If the data in statement I alone are sufficient to answer the question, while the data in statement II alone are not sufficient to answer the question
  • (B) If the data in statement II alone are sufficient to answer the question, while the data in statement I alone are not sufficient to answer the question
  • (C) If the data either in statement I alone or in statement II alone are sufficient to answer the question
  • (D) If the data given in both statements I and II together are not sufficient to answer the question and
  • (E) If the data in both statements I and II together are necessary to answer the question.
Question -
Question: How many children are there in the row of children facing North ?
Statements:
I.
Vishakha who is fifth from the left end is eighth to the left of Ashish who is twelfth from the right end.

II.Rohit is fifth to the left of Nisha who is seventh from the right end and eighteenth from the left end.
  • a)
    I alone is sufficient while II alone is not sufficient
  • b)
    II alone is sufficient while I alone is not sufficient
  • c)
    Either I or II is sufficient
  • d)
    Neither I nor II is sufficient
  • e)
    Both I and II are sufficient
Correct answer is option 'C'. Can you explain this answer?

Arya Mehta answered
Since 8th to the left of 12th from the right is 20th from the right, so from I, we know that Vishakha is 5th from left and 20th from right i.e. there are 4 children to the left and 19 to the right of Vishakha. So, there are (4 + 1 + 19) i.e. 24 children in the row.
From II, Nisha is 7th from right and 18th from left end of the row.
So, there are (6 + 1 + 17) = 24 children in the row.

Directions to Solve
In each of the questions below consists of a question and two statements numbered I and II given below it. You have to decide whether the data provided in the statements are sufficient to answer the question. Read both the statements and
Give answer
  • (A) If the data in statement I alone are sufficient to answer the question, while the data in statement II alone are not sufficient to answer the question
  • (B) If the data in statement II alone are sufficient to answer the question, while the data in statement I alone are not sufficient to answer the question
  • (C) If the data either in statement I alone or in statement II alone are sufficient to answer the question
  • (D) If the data given in both statements I and II together are not sufficient to answer the question and
  • (E) If the data in both statements I and II together are necessary to answer the question.
 
Question -
 
Question: How is T related to K?
Statements:
I.
R's sister J has married Ts brother L, who is the only son of his parents.

II.K is the only daughter of L and J.
 
 
  • a)
    I alone is sufficient while II alone is not sufficient
  • b)
    II alone is sufficient while I alone is not sufficient
  • c)
    Either I or II is sufficient
  • d)
    Neither I nor II is sufficient
  • e)
    Both I and II are sufficient
Correct answer is option 'C'. Can you explain this answer?

Dhruv Mehra answered
The correct answer is E.
From I, we know that L is T's brother and J's husband. Since L is the only son of his parents, T is L's sister.
From II, we know that K is L's daughter.
Thus, from I and II, we conclude that T is the sister of K's father i.e. T is K's aunt.

Directions to Solve
In each of the questions below consists of a question and two statements numbered I and II given below it. You have to decide whether the data provided in the statements are sufficient to answer the question. Read both the statements and
Give answer
  • (A) If the data in statement I alone are sufficient to answer the question, while the data in statement II alone are not sufficient to answer the question
  • (B) If the data in statement II alone are sufficient to answer the question, while the data in statement I alone are not sufficient to answer the question
  • (C) If the data either in statement I alone or in statement II alone are sufficient to answer the question
  • (D) If the data given in both statements I and II together are not sufficient to answer the question and
  • (E) If the data in both statements I and II together are necessary to answer the question.
 
Question -
 
Question: On which day in April is Gautam's birthday?
Statements:
I.
Gautam was born exactly 28 years after his mother was born.

II.His mother will be 55 years 4 months and 5 days on August 18 this year.
 
 
  • a)
    I alone is sufficient while II alone is not sufficient
  • b)
    II alone is sufficient while I alone is not sufficient
  • c)
    Either I or II is sufficient
  • d)
    Neither I nor II is sufficient
  • e)
    Both I and II are sufficient
Correct answer is option 'E'. Can you explain this answer?

Clearly, the birthday of Gautam's mother can be found out from II and then Gautam's birthday can be determined using the fact given in I.

Directions to Solve
In each of the questions below consists of a question and two statements numbered I and II given below it. You have to decide whether the data provided in the statements are sufficient to answer the question. Read both the statements and
Give answer
  • (A) If the data in statement I alone are sufficient to answer the question, while the data in statement II alone are not sufficient to answer the question
  • (B) If the data in statement II alone are sufficient to answer the question, while the data in statement I alone are not sufficient to answer the question
  • (C) If the data either in statement I alone or in statement II alone are sufficient to answer the question
  • (D) If the data given in both statements I and II together are not sufficient to answer the question and
  • (E) If the data in both statements I and II together are necessary to answer the question.
Question -
Question: On which day of the week was birthday of Sahil ?
Statements:
I.
Sahil celebrated his birthday the very next day on which Arun celebrated his birthday.

II.The sister of Sahil was born on the third day of the week and two days after Sahil was born.
  • a)
    I alone is sufficient while II alone is not sufficient
  • b)
    II alone is sufficient while I alone is not sufficient
  • c)
    Either I or II is sufficient
  • d)
    Neither I nor II is sufficient
  • e)
    Both I and II are sufficient
Correct answer is option 'B'. Can you explain this answer?

Anand Goyal answered
I does not mention the day of the week on the birthday of either Arun or Sahil.
According to II, Sahil's sister was born on Wednesday and Sahil was born two days before Wednesday i.e. on Monday.

Directions to Solve
In each of the questions below consists of a question and two statements numbered I and II given below it. You have to decide whether the data provided in the statements are sufficient to answer the question. Read both the statements and
Give answer
  • (A) If the data in statement I alone are sufficient to answer the question, while the data in statement II alone are not sufficient to answer the question
  • (B) If the data in statement II alone are sufficient to answer the question, while the data in statement I alone are not sufficient to answer the question
  • (C) If the data either in statement I alone or in statement II alone are sufficient to answer the question
  • (D) If the data given in both statements I and II together are not sufficient to answer the question and
  • (E) If the data in both statements I and II together are necessary to answer the question.
Question -
Question: Who is to the immediate right of P among five persons P, Q, R, S and T facing North ?
Statements:
I.
R is third to the left of Q and P is second to the right of R.

II.Q is to the immediate left of T who is second to the right of P.
  • a)
    I alone is sufficient while II alone is not sufficient
  • b)
    II alone is sufficient while I alone is not sufficient
  • c)
    Either I or II is sufficient
  • d)
    Neither I nor II is sufficient
  • e)
    Both I and II are sufficient
Correct answer is option 'C'. Can you explain this answer?

Niti Iyer answered
From I, we have the order: R, -, P, Q.
From II, we have the order: P, Q, T.
Clearly, each one of the above two orders indicates that Q is to the immediate right of P.

Directions to Solve
In each of the questions below consists of a question and two statements numbered I and II given below it. You have to decide whether the data provided in the statements are sufficient to answer the question. Read both the statements and
Give answer
  • (A) If the data in statement I alone are sufficient to answer the question, while the data in statement II alone are not sufficient to answer the question
  • (B) If the data in statement II alone are sufficient to answer the question, while the data in statement I alone are not sufficient to answer the question
  • (C) If the data either in statement I alone or in statement II alone are sufficient to answer the question
  • (D) If the data given in both statements I and II together are not sufficient to answer the question and
  • (E) If the data in both statements I and II together are necessary to answer the question.
 
Question -
 
Question: What is Nitin's rank from the top in a class of forty students ?
Statements:
I.
There are ten students between Nitin and Deepak.

II.Deepak is twentieth from the top.
 
 
  • a)
    I alone is sufficient while II alone is not sufficient
  • b)
    II alone is sufficient while I alone is not sufficient
  • c)
    Either I or II is sufficient
  • d)
    Neither I nor II is sufficient
  • e)
    Both I and II are sufficient
Correct answer is option 'D'. Can you explain this answer?

Maulik Roy answered
Since there are ten students between Nitin and Deepak, so Nitin may be eleven ranks above or below Deepak. Thus, Nitin may be 9th or 31st from the top.

Directions to Solve
In each of the questions below consists of a question and two statements numbered I and II given below it. You have to decide whether the data provided in the statements are sufficient to answer the question. Read both the statements and
Give answer
  • (A) If the data in statement I alone are sufficient to answer the question, while the data in statement II alone are not sufficient to answer the question
  • (B) If the data in statement II alone are sufficient to answer the question, while the data in statement I alone are not sufficient to answer the question
  • (C) If the data either in statement I alone or in statement II alone are sufficient to answer the question
  • (D) If the data given in both statements I and II together are not sufficient to answer the question and
  • (E) If the data in both statements I and II together are necessary to answer the question.
Question -
Question: How many students in a class play football ?
Statements:
I.
Only boys play football.

II.There are forty boys and thirty girls in the class.
  • a)
    I alone is sufficient while II alone is not sufficient
  • b)
    II alone is sufficient while I alone is not sufficient
  • c)
    Either I or II is sufficient
  • d)
    Neither I nor II is sufficient
  • e)
    Both I and II are sufficient
Correct answer is option 'D'. Can you explain this answer?

Ritika Yadav answered
Statement Analysis:

Statement I:
- Only boys play football.
- This statement alone does not provide a specific number of students playing football since the total number of boys in the class is not given.

Statement II:
- There are forty boys and thirty girls in the class.
- This statement alone does not provide the information on how many students play football as it does not specify whether girls play football or not.

Combined Analysis:
- Combining both statements, we know that there are forty boys in the class, but we do not know if all boys play football.
- Additionally, we are not provided any information about the girls playing football.
- Therefore, even when the two statements are considered together, we cannot determine the exact number of students who play football.
Therefore, the correct answer is option D) Neither I nor II is sufficient.

Directions to Solve
In each of the questions below consists of a question and two statements numbered I and II given below it. You have to decide whether the data provided in the statements are sufficient to answer the question. Read both the statements and
Give answer
  • (A) If the data in statement I alone are sufficient to answer the question, while the data in statement II alone are not sufficient to answer the question
  • (B) If the data in statement II alone are sufficient to answer the question, while the data in statement I alone are not sufficient to answer the question
  • (C) If the data either in statement I alone or in statement II alone are sufficient to answer the question
  • (D) If the data given in both statements I and II together are not sufficient to answer the question and
  • (E) If the data in both statements I and II together are necessary to answer the question.
 
Question -
 
Question: How much was the total sale of the company ?
Statements:
I.
The company sold 8000 units of product A each costing Rs. 25.

II.This company has no other product line.
 
 
  • a)
    I alone is sufficient while II alone is not sufficient
  • b)
    II alone is sufficient while I alone is not sufficient
  • c)
    Either I or II is sufficient
  • d)
    Neither I nor II is sufficient
  • e)
    Both I and II are sufficient
Correct answer is option 'C'. Can you explain this answer?

Dhruv Mehra answered
The correct answer is E.
From I, total sale of product A = Rs. (8000 x 25) = Rs. 200000.
From II, we know that the company deals only in product A.
This implies that sale of product A is the total sale of the company, which is Rs. 200000.

Directions to Solve

In each of the questions below consists of a question and two statements numbered I and II given below it. You have to decide whether the data provided in the statements are sufficient to answer the question. Read both the statements and

Give answer

  • (A) If the data in statement I alone are sufficient to answer the question, while the data in statement II alone are not sufficient to answer the question
  • (B) If the data in statement II alone are sufficient to answer the question, while the data in statement I alone are not sufficient to answer the question
  • (C) If the data either in statement I alone or in statement II alone are sufficient to answer the question
  • (D) If the data given in both statements I and II together are not sufficient to answer the question and
  • (E) If the data in both statements I and II together are necessary to answer the question.

Question -

Question: On a T.V. channel, four serials A, B, C, and D were screened, one on eacn days, on four consecutive days but not necessarily in that order. On which day was serial C screened?

Statements:
I. The first serial was screened on 23rd, Tuesday and was followed by serial D.
II. Serial A was not screened on 25th and one serial was screened between serials A and B.

 
 
  • a)
    I alone is sufficient while II alone is not sufficient
  • b)
    II alone is sufficient while I alone is not sufficient
  • c)
    Both I and II are sufficient
  • d)
    Neither I nor II is sufficient
  • e)
    Either I or II is sufficient
Correct answer is option 'C'. Can you explain this answer?

Manasa Sarkar answered
From I, we know that the serials were screened on 23rd, 24th, 25th and 26th.
Clearly, D was screened second i.e. on 24th, Wednesday.
From II, we know that one serial was screened between A and B.
So, A and B were screened first and third, i.e. on 23rd and 25th. But, A was not screened on 25th.
So, A was screened on 23rd and B on 25th. Thus, C was screened on 26th, Friday.

Directions to Solve
In each of the questions below consists of a question and two statements numbered I and II given below it. You have to decide whether the data provided in the statements are sufficient to answer the question. Read both the statements and
Give answer
  • (A) If the data in statement I alone are sufficient to answer the question, while the data in statement II alone are not sufficient to answer the question
  • (B) If the data in statement II alone are sufficient to answer the question, while the data in statement I alone are not sufficient to answer the question
  • (C) If the data either in statement I alone or in statement II alone are sufficient to answer the question
  • (D) If the data given in both statements I and II together are not sufficient to answer the question and
  • (E) If the data in both statements I and II together are necessary to answer the question.
 
Question -
 
Question: How many sons does D have ?
Statements:
A's father has three children.

II.B is A's brother and son of D.
 
 
  • a)
    I alone is sufficient while II alone is not sufficient
  • b)
    II alone is sufficient while I alone is not sufficient
  • c)
    Either I or II is sufficient
  • d)
    Neither I nor II is sufficient
  • e)
    Both I and II are sufficient
Correct answer is option 'D'. Can you explain this answer?

From both I and II together, we can conclude that A and B are the children of D, but the sex of A and the third child of D is not known. So, both I and II together are also not sufficient to answer the question.

Directions to Solve
In each of the questions below consists of a question and two statements numbered I and II given below it. You have to decide whether the data provided in the statements are sufficient to answer the question. Read both the statements and
Give answer
  • (A) If the data in statement I alone are sufficient to answer the question, while the data in statement II alone are not sufficient to answer the question
  • (B) If the data in statement II alone are sufficient to answer the question, while the data in statement I alone are not sufficient to answer the question
  • (C) If the data either in statement I alone or in statement II alone are sufficient to answer the question
  • (D) If the data given in both statements I and II together are not sufficient to answer the question and
  • (E) If the data in both statements I and II together are necessary to answer the question.
 
Question -
 
Question: In a row of five buildings - P, Q, R, S and T, which building is in the middle ?
Statements:
I.
Buildings S and Q are at the two extreme ends of the row.

II.Building ,T is to the right of building R.
 
 
  • a)
    I alone is sufficient while II alone is not sufficient
  • b)
    II alone is sufficient while I alone is not sufficient
  • c)
    Either I or II is sufficient
  • d)
    Neither I nor II is sufficient
  • e)
    Both I and II are sufficient
Correct answer is option 'D'. Can you explain this answer?

Alok Verma answered
From I, we have the order : S, -, -, -, Q. From II, we have the order : R, T. Combining the above two, we get two possible orders : S, R, T, P, Q or S, P, R, T, Q. Thus, either T or R is in the middle.

Directions to Solve
In each of the questions below consists of a question and two statements numbered I and II given below it. You have to decide whether the data provided in the statements are sufficient to answer the question. Read both the statements and
Give answer
  • (A) If the data in statement I alone are sufficient to answer the question, while the data in statement II alone are not sufficient to answer the question
  • (B) If the data in statement II alone are sufficient to answer the question, while the data in statement I alone are not sufficient to answer the question
  • (C) If the data either in statement I alone or in statement II alone are sufficient to answer the question
  • (D) If the data given in both statements I and II together are not sufficient to answer the question and
  • (E) If the data in both statements I and II together are necessary to answer the question.
Question -
Question: On which date of the month was Anjali born in February 2004 ?
Statements:
I.
Anjali was born on an even date of the month.

II.Anjali's birth date was a prime number.
  • a)
    I alone is sufficient while II alone is not sufficient
  • b)
    II alone is sufficient while I alone is not sufficient
  • c)
    Either I or II is sufficient
  • d)
    Neither I nor II is sufficient
  • e)
    Both I and II are sufficient
Correct answer is option 'C'. Can you explain this answer?

Gayatri Sarkar answered
From I and II, we conclude that Anjali was born in February 2004 on a date which is an even prime number. Since the only even prime number is 2, so Anjali was born on 2nd February, 2004.

Directions to Solve
In each of the questions below consists of a question and two statements numbered I and II given below it. You have to decide whether the data provided in the statements are sufficient to answer the question. Read both the statements and
Give answer
  • (A) If the data in statement I alone are sufficient to answer the question, while the data in statement II alone are not sufficient to answer the question
  • (B) If the data in statement II alone are sufficient to answer the question, while the data in statement I alone are not sufficient to answer the question
  • (C) If the data either in statement I alone or in statement II alone are sufficient to answer the question
  • (D) If the data given in both statements I and II together are not sufficient to answer the question and
  • (E) If the data in both statements I and II together are necessary to answer the question.
 
Question -
 
Question: How many visitors saw the exhibition yesterday ?
Statements:
I.
Each entry pass holder can take up to three persons with him/her.

II.In all, 243 passes were sold yesterday.
 
 
  • a)
    I alone is sufficient while II alone is not sufficient
  • b)
    II alone is sufficient while I alone is not sufficient
  • c)
    Either I or II is sufficient
  • d)
    Neither I nor II is sufficient
  • e)
    Both I and II are sufficient
Correct answer is option 'D'. Can you explain this answer?

Saanvi Rane answered
Statement Analysis:

Statement I: Each entry pass holder can take up to three persons with him/her.
- This statement provides information about the number of people allowed per entry pass holder, but it does not give the total number of visitors.

Statement II: In all, 243 passes were sold yesterday.
- This statement gives the total number of passes sold, but it does not specify how many people actually visited the exhibition.

Combined Analysis:
- Statement I gives the limit on the number of people per pass holder, but it does not give the actual number of pass holders.
- Statement II provides the total number of passes sold, but it does not specify how many people each pass holder brought with them.
- Without knowing the exact number of pass holders and how many people each pass holder brought, we cannot determine the total number of visitors.
- Therefore, the data given in both statements I and II together are not sufficient to answer the question.
- The correct answer is option (D) Neither I nor II is sufficient.

Directions to Solve: In each of the questions below consists of a question and two statements numbered I and II given below it. You have to decide whether the data provided in the statements are sufficient to answer the question. Read both the statements and
Give answer:
- (A) If the data in statement I alone are sufficient to answer the question, while the data in statement II alone are not sufficient to answer the question
- (B) If the data in statement II alone are sufficient to answer the question, while the data in statement I alone are not sufficient to answer the question
- (C) If the data either in statement I alone or in statement II alone are sufficient to answer the question
- (D) If the data given in both statements I and II together are not sufficient to answer the question and
- (E) If the data in both statements I and II together are necessary to answer the question.
Question: Which word in the code language means 'flower' ?
Statements:
I.
'de fu la pane' means 'rose flower is beautiful' and 'la quiz' means 'beautiful tree'.

II.'de la chin' means 'red rose flower' and 'pa chin' means 'red tea'.
  • a)
    I alone is sufficient while II alone is not sufficient
  • b)
    Neither I nor II is sufficient
  • c)
    Either I or II is sufficient
  • d)
    II alone is sufficient while I alone is not sufficient
  • e)
    Both I and II are sufficient
Correct answer is option 'D'. Can you explain this answer?

Deepak Basu answered
Understanding the Question
The question asks which word in a coded language means "flower". We are given two statements to analyze the available information regarding the codes for various words.
Analyzing Statement I
- Statement I: "de fu la pane" means "rose flower is beautiful" and "la quiz" means "beautiful tree".
From this statement:
- We know that "la" means "beautiful" (common in both phrases).
- The word "fu" is likely related to "rose" since it appears in the context of flowers.
- However, we do not have explicit information about the word for "flower".
This means Statement I alone is not sufficient to determine which word means "flower".
Analyzing Statement II
- Statement II: "de la chin" means "red rose flower" and "pa chin" means "red tea".
From this statement:
- The word "chin" is common, indicating it means "red".
- Here, "de" is likely "flower" because it connects with "rose" in the phrase "red rose flower".
Thus, Statement II alone provides sufficient information to conclude that "de" means "flower".
Conclusion
- Statement I alone does not provide enough information about the word for "flower".
- Statement II alone is sufficient, as it directly includes the word associated with "flower".
Therefore, the answer is option 'D': Statement II alone is sufficient while Statement I alone is not sufficient.

Directions to Solve
In each of the questions below consists of a question and two statements numbered I and II given below it. You have to decide whether the data provided in the statements are sufficient to answer the question. Read both the statements and
Give answer
  • (A) If the data in statement I alone are sufficient to answer the question, while the data in statement II alone are not sufficient to answer the question
  • (B) If the data in statement II alone are sufficient to answer the question, while the data in statement I alone are not sufficient to answer the question
  • (C) If the data either in statement I alone or in statement II alone are sufficient to answer the question
  • (D) If the data given in both statements I and II together are not sufficient to answer the question and
  • (E) If the data in both statements I and II together are necessary to answer the question.
 
Question -
 
Question: How is Sulekha related to Nandini ?
Statements:
I.
Sulekha's husband is the only son of Nandini's mother.

II.Sulekha's brother and Nandini's husband are cousins.
 
 
  • a)
    I alone is sufficient while II alone is not sufficient
  • b)
    II alone is sufficient while I alone is not sufficient
  • c)
    Either I or II is sufficient
  • d)
    Neither I nor II is sufficient
  • e)
    Both I and II are sufficient
Correct answer is option 'C'. Can you explain this answer?

Aman Roy answered
Statement Analysis:

Statement I:
- Sulekha's husband is the only son of Nandini's mother.
- From this statement, we can infer that Sulekha is Nandini's sister-in-law as her husband is the only son of Nandini's mother.

Statement II:
- Sulekha's brother and Nandini's husband are cousins.
- This statement implies that Sulekha and Nandini are related through marriage, as Sulekha's brother is related to Nandini's husband.

Explanation:
- Combining both statements, it is clear that Sulekha is Nandini's sister-in-law, and they are related through marriage.
- Therefore, either Statement I or Statement II alone is sufficient to determine that Sulekha is related to Nandini.
- Hence, the correct answer is option (C) either I or II is sufficient.

Directions to Solve
In each of the questions below consists of a question and two statements numbered I and II given below it. You have to decide whether the data provided in the statements are sufficient to answer the question. Read both the statements and
Give answer
  • (A) If the data in statement I alone are sufficient to answer the question, while the data in statement II alone are not sufficient to answer the question
  • (B) If the data in statement II alone are sufficient to answer the question, while the data in statement I alone are not sufficient to answer the question
  • (C) If the data either in statement I alone or in statement II alone are sufficient to answer the question
  • (D) If the data given in both statements I and II together are not sufficient to answer the question and
  • (E) If the data in both statements I and II together are necessary to answer the question.
 
Question -
 
Question: On which day of the week did Hitesh visit the zoo ?
Statements:
I.
Hitesh did not visit zoo either on Tuesday or on Thursday.

II.Hitesh visited zoo two days before his mother reached his house which was day after Monday.
 
 
  • a)
    I alone is sufficient while II alone is not sufficient
  • b)
    II alone is sufficient while I alone is not sufficient
  • c)
    Either I or II is sufficient
  • d)
    Neither I nor II is sufficient
  • e)
    Both I and II are sufficient
Correct answer is option 'B'. Can you explain this answer?

Janhavi Chopra answered
According to I, Hitesh visited the zoo on any of the week days except Tuesday and Thursday.
According to II, Hitesh's mother reached his house day after Monday i.e. on Tuesday.
Thus, Hitesh visited zoo two days before Tuesday i.e. on Sunday.

Directions to Solve
In each of the questions below consists of a question and two statements numbered I and II given below it. You have to decide whether the data provided in the statements are sufficient to answer the question. Read both the statements and
Give answer
  • (A) If the data in statement I alone are sufficient to answer the question, while the data in statement II alone are not sufficient to answer the question
  • (B) If the data in statement II alone are sufficient to answer the question, while the data in statement I alone are not sufficient to answer the question
  • (C) If the data either in statement I alone or in statement II alone are sufficient to answer the question
  • (D) If the data given in both statements I and II together are not sufficient to answer the question and
  • (E) If the data in both statements I and II together are necessary to answer the question.
 
Question -
 
Question: Among A, B, C, D, E and F, who is the heaviest ?
Statements:
I.
A and D are heavier than B, E and F but none of them is the heaviest.

II.A is heavier than D but lighter than C.
 
 
  • a)
    I alone is sufficient while II alone is not sufficient
  • b)
    II alone is sufficient while I alone is not sufficient
  • c)
    Either I or II is sufficient
  • d)
    Neither I nor II is sufficient
  • e)
    Both I and II are sufficient
Correct answer is option 'A'. Can you explain this answer?

Athira Sen answered
From I, we conclude that since none of A and D is the heaviest and each one of B, E and F is lighter than both A and D, so C is the heaviest.

Directions to Solve: In each of the questions below consists of a question and two statements numbered I and II given below it. You have to decide whether the data provided in the statements are sufficient to answer the question. Read both the statements and Give answer: -
(A) If the data in statement I alone are sufficient to answer the question, while the data in statement II alone are not sufficient to answer the question -
(B) If the data in statement II alone are sufficient to answer the question, while the data in statement I alone are not sufficient to answer the question -
(C) If the data either in statement I alone or in statement II alone are sufficient to answer the question -
(D) If the data given in both statements I and II together are not sufficient to answer the question and -
(E) If the data in both statements I and II together are necessary to answer the question.
Question: What is the rank of K from bottom in a class of 30 students?
Statements:
I.M is third from the top and there are five students between M and K.
II.The rank of P is fourth from the bottom and there are 17 students between P and K.
  • a)
    Data in Statement I alone are sufficient to answer the question, while the data in Statement II alone are not sufficient to answer the question.
  • b)
    Data in Statement II alone are sufficient to answer the question, while the data in Statement I alone are not sufficient to answer the question.
  • c)
    Data either in Statement I alone or in Statement II alone are sufficient to answer the question.
  • d)
    Data in both the Statements I and II together are not sufficient to answer the question.
  • e)
    Data in both the Statements I and II together are necessary to answer the question.
Correct answer is option 'C'. Can you explain this answer?

Directions to Solve: In each of the questions below consists of a question and two statements numbered I and II given below it. You have to decide whether the data provided in the statements are sufficient to answer the question. Read both the statements and
Give answer:
- (A) If the data in statement I alone are sufficient to answer the question, while the data in statement II alone are not sufficient to answer the question
- (B) If the data in statement II alone are sufficient to answer the question, while the data in statement I alone are not sufficient to answer the question
- (C) If the data either in statement I alone or in statement II alone are sufficient to answer the question
- (D) If the data given in both statements I and II together are not sufficient to answer the question and
- (E) If the data in both statements I and II together are necessary to answer the question.
Question: How is X related to Y ?
Statements:
I.
Y says, "I have only one brother".

II.X says, "I have only one sister".
  • a)
    I alone is sufficient while II alone is not sufficient
  • b)
    II alone is sufficient while I alone is not sufficient
  • c)
    Either I or II is sufficient
  • d)
    Neither I nor II is sufficient
  • e)
    Both I and II are sufficient
Correct answer is option 'D'. Can you explain this answer?

Explanation:

Statement I:
Y says, "I have only one brother."
- This statement indicates that Y is male and has only one brother. However, it does not provide any information about X or the relationship between X and Y.

Statement II:
X says, "I have only one sister."
- This statement indicates that X is female and has only one sister. Similar to Statement I, this statement alone does not provide any information about Y or the relationship between X and Y.

Combined:
- When we combine both statements, we can deduce that X is female and has only one sister, while Y is male and has only one brother. However, we still cannot determine the exact relationship between X and Y. They could be siblings, cousins, or even unrelated.
Therefore, the data given in both statements I and II together are not sufficient to answer the question of how X is related to Y. Hence, the correct answer is option (D) Neither I nor II is sufficient.

Directions to Solve
In each of the questions below consists of a question and two statements numbered I and II given below it. You have to decide whether the data provided in the statements are sufficient to answer the question. Read both the statements and
Give answer
  • (A) If the data in statement I alone are sufficient to answer the question, while the data in statement II alone are not sufficient to answer the question
  • (B) If the data in statement II alone are sufficient to answer the question, while the data in statement I alone are not sufficient to answer the question
  • (C) If the data either in statement I alone or in statement II alone are sufficient to answer the question
  • (D) If the data given in both statements I and II together are not sufficient to answer the question and
  • (E) If the data in both statements I and II together are necessary to answer the question.
 
Question -
 
Question: In a certain code, 'nop al ed' means They like flowers'. Which code word means 'flowers' ?
Statements:
I.
'id nim nop' means 'They are innocent'.

II.'gob ots al' means 'We like roses'.
 
 
  • a)
    I alone is sufficient while II alone is not sufficient
  • b)
    II alone is sufficient while I alone is not sufficient
  • c)
    Either I or II is sufficient
  • d)
    Neither I nor II is sufficient
  • e)
    Both I and II are sufficient
Correct answer is option 'E'. Can you explain this answer?

Shreya Rane answered
Analysis:

Statement I:
- The code "nop" means "They".
- The code "al" means "like".
- The code "ed" means "flowers".
- From this statement alone, we can determine the meaning of the word "flowers" as "ed".

Statement II:
- The code "al" means "like".
- From this statement alone, we can determine the meaning of the word "flowers" as "roses".

Combined Analysis:
- By combining both statements, we can see that the word "al" has the same meaning in both statements, which is "like".
- However, statement I provides the code for "flowers" as "ed", while statement II does not provide the code for "flowers".
- Therefore, we need both statements together to determine that the code word for "flowers" is "ed".
Therefore, the correct answer is (E) Both I and II are sufficient.

Directions to Solve
In each of the questions below consists of a question and two statements numbered I and II given below it. You have to decide whether the data provided in the statements are sufficient to answer the question. Read both the statements and
Give answer
  • (A) If the data in statement I alone are sufficient to answer the question, while the data in statement II alone are not sufficient to answer the question
  • (B) If the data in statement II alone are sufficient to answer the question, while the data in statement I alone are not sufficient to answer the question
  • (C) If the data either in statement I alone or in statement II alone are sufficient to answer the question
  • (D) If the data given in both statements I and II together are not sufficient to answer the question and
  • (E) If the data in both statements I and II together are necessary to answer the question.
 
Question -
 
Question: How is M related to N?
Statements:
I.
P, who has only two kids, M and N, is the mother-in-law of Q, who is sister-in-law of N.

II.R, the sister-in-law of M, is the daughter-in-law of S, who has only two kids, M and N.
 
 
  • a)
    I alone is sufficient while II alone is not sufficient
  • b)
    II alone is sufficient while I alone is not sufficient
  • c)
    Either I or II is sufficient
  • d)
    Neither I nor II is sufficient
  • e)
    Both I and II are sufficient
Correct answer is option 'A'. Can you explain this answer?

Mainak Malik answered
From I, we conclude that P is the mother of M and N, while Q is the daughter-in-law of P and sister-in-law of N. Thus, Q is M's wife and hence, M is N's brother.
From II, we conclude that M and N are the children of S. Also, R is the daughter-in-law of S and sister-in-law of M. So, R is N's wife and thus, N is M's brother. Hence, M is either brother or sister of N.

Directions to Solve
In each of the questions below consists of a question and two statements numbered I and II given below it. You have to decide whether the data provided in the statements are sufficient to answer the question. Read both the statements and
Give answer
  • (A) If the data in statement I alone are sufficient to answer the question, while the data in statement II alone are not sufficient to answer the question
  • (B) If the data in statement II alone are sufficient to answer the question, while the data in statement I alone are not sufficient to answer the question
  • (C) If the data either in statement I alone or in statement II alone are sufficient to answer the question
  • (D) If the data given in both statements I and II together are not sufficient to answer the question and
  • (E) If the data in both statements I and II together are necessary to answer the question.
Question: The last Sunday of March, 2006 fell on which date ?
Statements:
I.
The first Sunday of that month fell on 5th.

II.The last day of that month was Friday.
  • a)
    I alone is sufficient while II alone is not sufficient
  • b)
    II alone is sufficient while I alone is not sufficient
  • c)
    Either I or II is sufficient
  • d)
    Neither I nor II is sufficient
  • e)
    Both I and II are sufficient
Correct answer is option 'C'. Can you explain this answer?

Madhurima Dey answered
From I, we conclude that 5th, 12th, 19th and 26th of March, 2006 were Sundays.
So, the last Sunday fell on 26th.
From II, we conclude that 31st March, 2006 was Friday. Thus, 26th March, 2006 was the last Sunday of the month.

Directions to Solve
In each of the questions below consists of a question and two statements numbered I and II given below it. You have to decide whether the data provided in the statements are sufficient to answer the question. Read both the statements and
Give answer
  • (A) If the data in statement I alone are sufficient to answer the question, while the data in statement II alone are not sufficient to answer the question
  • (B) If the data in statement II alone are sufficient to answer the question, while the data in statement I alone are not sufficient to answer the question
  • (C) If the data either in statement I alone or in statement II alone are sufficient to answer the question
  • (D) If the data given in both statements I and II together are not sufficient to answer the question and
  • (E) If the data in both statements I and II together are necessary to answer the question.
 
Question -
 
Question: Among M, N, D, P and K, who earns more than only the least earner among them ?
Statements:
I.
N earns more than M and P but less than only D.

II.M earns more than P who earns less than K.
 
 
  • a)
    I alone is sufficient while II alone is not sufficient
  • b)
    II alone is sufficient while I alone is not sufficient
  • c)
    Either I or II is sufficient
  • d)
    Neither I nor II is sufficient
  • e)
    Both I and II are sufficient
Correct answer is option 'D'. Can you explain this answer?

From I, we have: N > M, N > P, D > N. Thus, we have: D > N > M > P or D>N> P>M.
But, from II, M earns more than P i.e. D > N > M > P. Also, since P earns less than K and N earns less than only D, so we have: D>N>K>M>P or D>N>M> K > P.
Hence, either K or M earns more than only the least earner i.e. P.

Directions to Solve
In each of the questions below consists of a question and two statements numbered I and II given below it. You have to decide whether the data provided in the statements are sufficient to answer the question. Read both the statements and
Give answer
  • (A) If the data in statement I alone are sufficient to answer the question, while the data in statement II alone are not sufficient to answer the question
  • (B) If the data in statement II alone are sufficient to answer the question, while the data in statement I alone are not sufficient to answer the question
  • (C) If the data either in statement I alone or in statement II alone are sufficient to answer the question
  • (D) If the data given in both statements I and II together are not sufficient to answer the question and
  • (E) If the data in both statements I and II together are necessary to answer the question.
 
Question -
 
Question: How is T related to K?
Statements:
I.
K has two sons; one of the sons is A.

II.The mother of T has only two sons - Aand B.
 
 
  • a)
    I alone is sufficient while II alone is not sufficient
  • b)
    II alone is sufficient while I alone is not sufficient
  • c)
    Either I or II is sufficient
  • d)
    Neither I nor II is sufficient
  • e)
    Both I and II are sufficient
Correct answer is option 'E'. Can you explain this answer?

From II, we know that T's mother has only two sons, A and B. This implies that T is the sister of both A and B. But, from I, A is also K's son. So, T is the daughter of K.

Given below is a question and two statements numbered I and II given below it. You have to decide whether the data provided in the statements is sufficient to answer the question. You should use the given data and your knowledge of Mathematics to choose between the possible answers.
Is a number T divisible by 144?
I. T is a multiple of 216.
II. T is divisible by square of cube of 2.
  • a)
    Choose this option if the Question can be answered by one of the Statements alone but not by the other.  
  • b)
    Choose this option if the Question can be answered by either Statement alone.
  • c)
    Choose this option if the Question can be answered by using both Statements together, but cannot be answered by using either Statement alone.  
  • d)
    Choose this option if the Question cannot be answered even by using both Statements together.
  • e)
    None of these
Correct answer is option 'C'. Can you explain this answer?

Vinod Mehta answered
From statement I:
If T is multiple of 216, then T can be 216 which is not divisible by 144 or T can be 432 which is divisible by 144. So, divisibility of T with 144 cannot be uniquely established.
∴ Statement I alone is not sufficient to answer the question.
From statement II:
T is divisible by square of cube of 2.
Square of cube of 2 = (2 × 2 × 2)2 = 64
⇒ T is a multiple of 64.
So, T can be 192 which is not divisible by 144 or T can be 576 which is divisible by 144. So, divisibility of T with 144 cannot be uniquely established.
∴ Statement II alone is not sufficient to answer the question.
From statements I and II together:
T is divisible by square of cube of 2.
Square of cube of 2 = (2 × 2 × 2)2 = 64
⇒ T is a multiple of 64.
Also, T is a multiple of 216.
⇒ T will be a multiple of LCM(216, 64).
⇒ T will be a multiple of 1728.
We know, 1728 = 144 x 12.
If T is divisible by 1728, it would be divisible by all its factors.
∴ T will be divisible by 144.
∴ Using both the statements together, we can answer the given question.

Directions to Solve
In each of the questions below consists of a question and two statements numbered I and II given below it. You have to decide whether the data provided in the statements are sufficient to answer the question. Read both the statements and
Give answer
  • (A) If the data in statement I alone are sufficient to answer the question, while the data in statement II alone are not sufficient to answer the question
  • (B) If the data in statement II alone are sufficient to answer the question, while the data in statement I alone are not sufficient to answer the question
  • (C) If the data either in statement I alone or in statement II alone are sufficient to answer the question
  • (D) If the data given in both statements I and II together are not sufficient to answer the question and
  • (E) If the data in both statements I and II together are necessary to answer the question.
 
Question -
 
Question: How many employees of Bank Z opted for VRS ?
Statements:
I.
18% of the 950 officer cadre employees and 6% of the 1100 other cadre employees opted for VRS.

II.28% of the employees in the age-group of 51 to 56 and 17% of the employees in all other age-groups opted for VRS.
 
 
  • a)
    I alone is sufficient while II alone is not sufficient
  • b)
    II alone is sufficient while I alone is not sufficient
  • c)
    Either I or II is sufficient
  • d)
    Neither I nor II is sufficient
  • e)
    Both I and II are sufficient
Correct answer is option 'A'. Can you explain this answer?

Nisha Dasgupta answered
From I, we have: number of employees who opted for VRS = 18% of 950 + 6% of 1100 = 171 + 66 = 237.
From II, we cannot get the required answer until and unless the number of employees in age-group 51 to 56 and other age-groups is known.

Directions to Solve
In each of the questions below consists of a question and two statements numbered I and II given below it. You have to decide whether the data provided in the statements are sufficient to answer the question. Read both the statements and
Give answer
  • (A) If the data in statement I alone are sufficient to answer the question, while the data in statement II alone are not sufficient to answer the question
  • (B) If the data in statement II alone are sufficient to answer the question, while the data in statement I alone are not sufficient to answer the question
  • (C) If the data either in statement I alone or in statement II alone are sufficient to answer the question
  • (D) If the data given in both statements I and II together are not sufficient to answer the question and
  • (E) If the data in both statements I and II together are necessary to answer the question.
 
Question -
 
Question: Among A, B, C, D and E, who is in the middle while standing in a row ?
Statements:
I.
C, who is third to the left of D, is to the immediate right of A and second to the left of E.

II.C is second to the left of E, who is not at any of the ends and who is third to the right of A. D is at one of the ends.
 
 
  • a)
    I alone is sufficient while II alone is not sufficient
  • b)
    II alone is sufficient while I alone is not sufficient
  • c)
    Either I or II is sufficient
  • d)
    Neither I nor II is sufficient
  • e)
    Both I and II are sufficient
Correct answer is option 'C'. Can you explain this answer?

From each one of I and II, we get the order : A, C, B, E, D. Clearly, B is in the middle.

Directions to Solve
In each of the questions below consists of a question and two statements numbered I and II given below it. You have to decide whether the data provided in the statements are sufficient to answer the question. Read both the statements and
Give answer
  • (A) If the data in statement I alone are sufficient to answer the question, while the data in statement II alone are not sufficient to answer the question
  • (B) If the data in statement II alone are sufficient to answer the question, while the data in statement I alone are not sufficient to answer the question
  • (C) If the data either in statement I alone or in statement II alone are sufficient to answer the question
  • (D) If the data given in both statements I and II together are not sufficient to answer the question and
  • (E) If the data in both statements I and II together are necessary to answer the question.
Question -
Question: What is the code for 'sky' in the code language ?
Statements:
I.
In the code language, 'sky is clear' is written as 'de ra fa'.

II.In the same code language, 'make it clear' is written as 'de ga jo'.
  • a)
    I alone is sufficient while II alone is not sufficient
  • b)
    II alone is sufficient while I alone is not sufficient
  • c)
    Either I or II is sufficient
  • d)
    Neither I nor II is sufficient
  • e)
    Both I and II are sufficient
Correct answer is option 'D'. Can you explain this answer?

Arshiya Bose answered
The only word common to I and II is 'clear' and as such, only the code for 'clear' can be ascertained from the given information.

Directions to Solve: In each of the questions below consists of a question and two statements numbered I and II given below it. You have to decide whether the data provided in the statements are sufficient to answer the question. Read both the statements and
Give answer:
- (A) If the data in statement I alone are sufficient to answer the question, while the data in statement II alone are not sufficient to answer the question
- (B) If the data in statement II alone are sufficient to answer the question, while the data in statement I alone are not sufficient to answer the question
- (C) If the data either in statement I alone or in statement II alone are sufficient to answer the question
- (D) If the data given in both statements I and II together are not sufficient to answer the question and
- (E) If the data in both statements I and II together are necessary to answer the question.
Question: How many doctors are practising in this town ?
Statements:
I.
There is one doctor per seven hundred residents.

II.There are 16 wards with each ward having as many doctors as the number of wards.
  • a)
    I alone is sufficient while II alone is not sufficient
  • b)
    II alone is sufficient while I alone is not sufficient
  • c)
    Either I or II is sufficient
  • d)
    Neither I nor II is sufficient
  • e)
    Both I and II are sufficient
Correct answer is option 'B'. Can you explain this answer?

From I, total number of doctors in town = (1/700 x N) , where N = total number of residents in town. But, the value of N is not known.
From II, total number of doctors in town
= (Number of wards in town) x (Number of doctors in each ward)
= 16 x 16 = 256.
So Option B is correct

Directions to Solve
In each of the questions below consists of a question and two statements numbered I and II given below it. You have to decide whether the data provided in the statements are sufficient to answer the question. Read both the statements and
Give answer
  • (A) If the data in statement I alone are sufficient to answer the question, while the data in statement II alone are not sufficient to answer the question
  • (B) If the data in statement II alone are sufficient to answer the question, while the data in statement I alone are not sufficient to answer the question
  • (C) If the data either in statement I alone or in statement II alone are sufficient to answer the question
  • (D) If the data given in both statements I and II together are not sufficient to answer the question and
  • (E) If the data in both statements I and II together are necessary to answer the question.
 
Question -
 
Question: How much money do Vivek and Suman have together ?
Statements:
I.
Suman has 20 rupees less than what Tarun has.

II.Vivek has 30 rupees more than what Tarun has.
 
 
  • a)
    I alone is sufficient while II alone is not sufficient
  • b)
    II alone is sufficient while I alone is not sufficient
  • c)
    Either I or II is sufficient
  • d)
    Neither I nor II is sufficient
  • e)
    Both I and II are sufficient
Correct answer is option 'D'. Can you explain this answer?

Manasa Sarkar answered
From I, we have: S = T - 20.
From II, we have: V = T + 30.
Thus, from both I and II, we have: V + S = (T + 30) + (T - 20) = (2 T + 10).
So, to get the required amount, we need to know the amount that Tarun has.

Directions to Solve
In each of the questions below consists of a question and two statements numbered I and II given below it. You have to decide whether the data provided in the statements are sufficient to answer the question. Read both the statements and
Give answer
  • (A) If the data in statement I alone are sufficient to answer the question, while the data in statement II alone are not sufficient to answer the question
  • (B) If the data in statement II alone are sufficient to answer the question, while the data in statement I alone are not sufficient to answer the question
  • (C) If the data either in statement I alone or in statement II alone are sufficient to answer the question
  • (D) If the data given in both statements I and II together are not sufficient to answer the question and
  • (E) If the data in both statements I and II together are necessary to answer the question.
 
Question -
 
Question: What is the code for 'or' in the code language?
Statements:
I.
'nik sa te' means 'right or wrong', 'ro da nik' means 'he is right' and 'fe te ro' means 'that is wrong'.

II.'pa nik la' means 'that right man', 'sa ne pa' means 'this or that' and 'ne ka re' means 'tell this there'.
 
 
  • a)
    I alone is sufficient while II alone is not sufficient
  • b)
    II alone is sufficient while I alone is not sufficient
  • c)
    Either I or II is sufficient
  • d)
    Neither I nor II is sufficient
  • e)
    Both I and II are sufficient
Correct answer is option 'C'. Can you explain this answer?

Lakshmi Nair answered
I. In 'right or wrong' and 'he is right', the common word is 'right' and the common code word is 'nik'. So 'nik' means 'right'. In 'right or wrong' and 'that is wrong', the common word is 'wrong' and the common code word is 'te'. So, 'te' means 'wrong'.
Thus, in 'right or wrong', 'sa' is the code for 'or'. II. In 'that right man' and 'this or that', the common word is 'that' and the common code word is 'pa'. So, 'pa' means 'that'. In 'this or that' and 'tell this there', the common word is 'this' and the common code word is 'ne'. So, 'ne' means 'this'. Thus, in 'this or that', 'sa' is the code for 'or'.

Directions to Solve
In each of the questions below consists of a question and two statements numbered I and II given below it. You have to decide whether the data provided in the statements are sufficient to answer the question. Read both the statements and
Give answer
  • (A) If the data in statement I alone are sufficient to answer the question, while the data in statement II alone are not sufficient to answer the question
  • (B) If the data in statement II alone are sufficient to answer the question, while the data in statement I alone are not sufficient to answer the question
  • (C) If the data either in statement I alone or in statement II alone are sufficient to answer the question
  • (D) If the data given in both statements I and II together are not sufficient to answer the question and
  • (E) If the data in both statements I and II together are necessary to answer the question.
 
Question -
 
Question: Who among P, Q, R, S and T is the lightest?
Statements:
I.
R is heavier than Q and T but lighter than S.

II.S is not the heaviest.
 
 
  • a)
    I alone is sufficient while II alone is not sufficient
  • b)
    II alone is sufficient while I alone is not sufficient
  • c)
    Either I or II is sufficient
  • d)
    Neither I nor II is sufficient
  • e)
    Both I and II are sufficient
Correct answer is option 'D'. Can you explain this answer?

Stuti Dasgupta answered
From I, we have: R>Q, R>T, S>R i.e. S>R>Q>T or S>R>T>Q.
From II, S is not the heaviest. So, P is the heaviest. Thus, we have: P > S > R > Q > T or P > S > R > T > Q. Hence, either T or Q is the lightest.

Directions to Solve
In each of the questions below consists of a question and two statements numbered I and II given below it. You have to decide whether the data provided in the statements are sufficient to answer the question. Read both the statements and
Give answer
  • (A) If the data in statement I alone are sufficient to answer the question, while the data in statement II alone are not sufficient to answer the question
  • (B) If the data in statement II alone are sufficient to answer the question, while the data in statement I alone are not sufficient to answer the question
  • (C) If the data either in statement I alone or in statement II alone are sufficient to answer the question
  • (D) If the data given in both statements I and II together are not sufficient to answer the question and
  • (E) If the data in both statements I and II together are necessary to answer the question.
 
Question -
 
Question: In a certain code language, '297' means 'tie clip button'. Which number means 'button' in that language ?
Statements:
I.
In that language, '926' means clip your tie'.

II.In that language, '175' means 'hole and button'.
 
 
  • a)
    I alone is sufficient while II alone is not sufficient
  • b)
    II alone is sufficient while I alone is not sufficient
  • c)
    Either I or II is sufficient
  • d)
    Neither I nor II is sufficient
  • e)
    Both I and II are sufficient
Correct answer is option 'C'. Can you explain this answer?

Nabanita Gupta answered
In given statement and I, the common words are 'tie' and 'clip' and the common codes are '2' and '9'. So, '2' and '9' are the codes for 'tie' and 'clip'. Thus, in the given statement, '7' means 'button'. In given statement and II, the common code word '7' represents the common word 'button'.

Directions to Solve
In each of the questions below consists of a question and two statements numbered I and II given below it. You have to decide whether the data provided in the statements are sufficient to answer the question. Read both the statements and
Give answer
  • (A) If the data in statement I alone are sufficient to answer the question, while the data in statement II alone are not sufficient to answer the question
  • (B) If the data in statement II alone are sufficient to answer the question, while the data in statement I alone are not sufficient to answer the question
  • (C) If the data either in statement I alone or in statement II alone are sufficient to answer the question
  • (D) If the data given in both statements I and II together are not sufficient to answer the question and
  • (E) If the data in both statements I and II together are necessary to answer the question.
 
Question -
 
Question: What is the numerical code for 'water' in a certain code ?
Statements:
I.
The code for 'give me water' is '719'.

II.The code for 'you can bring water for me' is written as '574186'.
 
 
  • a)
    I alone is sufficient while II alone is not sufficient
  • b)
    II alone is sufficient while I alone is not sufficient
  • c)
    Either I or II is sufficient
  • d)
    Neither I nor II is sufficient
  • e)
    Both I and II are sufficient
Correct answer is option 'D'. Can you explain this answer?

Jaya Gupta answered
In I and II, the common words are 'me' and 'water' and the common code numbers are '7' and '1'. So, the code for 'water' is either '7' or '1'.

Directions to Solve
In each of the questions below consists of a question and two statements numbered I and II given below it. You have to decide whether the data provided in the statements are sufficient to answer the question. Read both the statements and
Give answer
  • (A) If the data in statement I alone are sufficient to answer the question, while the data in statement II alone are not sufficient to answer the question
  • (B) If the data in statement II alone are sufficient to answer the question, while the data in statement I alone are not sufficient to answer the question
  • (C) If the data either in statement I alone or in statement II alone are sufficient to answer the question
  • (D) If the data given in both statements I and II together are not sufficient to answer the question and
  • (E) If the data in both statements I and II together are necessary to answer the question.
 
Question -
 
Question: How many gift boxes were sold on Monday ?
Statements:
I.
It was 10% more than the boxes sold on the earlier day i.e. Sunday.

II.Every third visitor to the shop purchased the box and 1500 visitors were there on Sunday
 
 
  • a)
    I alone is sufficient while II alone is not sufficient
  • b)
    II alone is sufficient while I alone is not sufficient
  • c)
    Either I or II is sufficient
  • d)
    Neither I nor II is sufficient
  • e)
    Both I and II are sufficient
Correct answer is option 'E'. Can you explain this answer?

Ankita Yadav answered
From II, we can conclude that (1500 * 3) = 500 boxes were sold on Sunday.
Thus, from I, we find that number of boxes sold on Monday = 500 + 10% of 500 = 550.

Directions to Solve
In each of the questions below consists of a question and two statements numbered I and II given below it. You have to decide whether the data provided in the statements are sufficient to answer the question. Read both the statements and
Give answer
  • (A) If the data in statement I alone are sufficient to answer the question, while the data in statement II alone are not sufficient to answer the question
  • (B) If the data in statement II alone are sufficient to answer the question, while the data in statement I alone are not sufficient to answer the question
  • (C) If the data either in statement I alone or in statement II alone are sufficient to answer the question
  • (D) If the data given in both statements I and II together are not sufficient to answer the question and
  • (E) If the data in both statements I and II together are necessary to answer the question.
 
Question -
 
Question: Who among P, Q, T, V and M is exactly in the middle when they are arranged in ascending order of their heights ?
Statements:
I.
V is taller than Q but shorter than M.

II.T is taller than Q and M but shorter than P.
 
 
  • a)
    I alone is sufficient while II alone is not sufficient
  • b)
    II alone is sufficient while I alone is not sufficient
  • c)
    Either I or II is sufficient
  • d)
    Neither I nor II is sufficient
  • e)
    Both I and II are sufficient
Correct answer is option 'E'. Can you explain this answer?

Jyoti Dasgupta answered
From I, we have: M > V > Q.
From II, we have: T > Q, T > M, P > T.
Combining the above two, we have: P>T>M>V>Q i.e. Q
Clearly, M is in the middle.

Directions to Solve
In each of the questions below consists of a question and two statements numbered I and II given below it. You have to decide whether the data provided in the statements are sufficient to answer the question. Read both the statements and
Give answer
  • (A) If the data in statement I alone are sufficient to answer the question, while the data in statement II alone are not sufficient to answer the question
  • (B) If the data in statement II alone are sufficient to answer the question, while the data in statement I alone are not sufficient to answer the question
  • (C) If the data either in statement I alone or in statement II alone are sufficient to answer the question
  • (D) If the data given in both statements I and II together are not sufficient to answer the question and
  • (E) If the data in both statements I and II together are necessary to answer the question.
 
Question -
 
Question: It is 8.00 p.m., when can Hemant get next bus for Ramnagar from Dhanpur?
Statements:
I.
Buses for Ramnagar leave after every 30 minutes, till 10 p.m.

II.Fifteen minutes ago, one bus has left for Ramnagar.
 
 
  • a)
    I alone is sufficient while II alone is not sufficient
  • b)
    II alone is sufficient while I alone is not sufficient
  • c)
    Either I or II is sufficient
  • d)
    Neither I nor II is sufficient
  • e)
    Both I and II are sufficient
Correct answer is option 'E'. Can you explain this answer?

Shivani Ahuja answered
II reveals that the previous bus had left at 7.45 p.m. As given in I, the next bus would leave after 30 minutes i.e. at 8.15 p.m.

Directions to Solve
In each of the questions below consists of a question and two statements numbered I and II given below it. You have to decide whether the data provided in the statements are sufficient to answer the question. Read both the statements and
Give answer
  • (A) If the data in statement I alone are sufficient to answer the question, while the data in statement II alone are not sufficient to answer the question
  • (B) If the data in statement II alone are sufficient to answer the question, while the data in statement I alone are not sufficient to answer the question
  • (C) If the data either in statement I alone or in statement II alone are sufficient to answer the question
  • (D) If the data given in both statements I and II together are not sufficient to answer the question and
  • (E) If the data in both statements I and II together are necessary to answer the question.
 
Question -
 
Question: In a row of five children A, B, C, D and E, who is standing in the middle ?
Statements:
I.
D is to the immediate right of E and B is to the immediate left of E.

II.B is at the extreme left of the, row.
 
 
  • a)
    I alone is sufficient while II alone is not sufficient
  • b)
    II alone is sufficient while I alone is not sufficient
  • c)
    Either I or II is sufficient
  • d)
    Neither I nor II is sufficient
  • e)
    Both I and II are sufficient
Correct answer is option 'E'. Can you explain this answer?

Uday Nambiar answered
From I, we have the order: B, E, D.
From II, B is at the extreme left of the row.
Thus, considering both I and II, we conclude that among the five children, D is the third and hence the middle child in the row.

Directions to Solve
In each of the questions below consists of a question and two statements numbered I and II given below it. You have to decide whether the data provided in the statements are sufficient to answer the question. Read both the statements and
Give answer
  • (A) If the data in statement I alone are sufficient to answer the question, while the data in statement II alone are not sufficient to answer the question
  • (B) If the data in statement II alone are sufficient to answer the question, while the data in statement I alone are not sufficient to answer the question
  • (C) If the data either in statement I alone or in statement II alone are sufficient to answer the question
  • (D) If the data given in both statements I and II together are not sufficient to answer the question and
  • (E) If the data in both statements I and II together are necessary to answer the question.
 
Question -
 
Question: What is the colour of the fresh grass ?
Statements:
I.
Blue is called green, red is called orange, orange is called yellow.

II.Yellow is called white, white is called black, green is called brown and brown is called purple.
 
 
  • a)
    I alone is sufficient while II alone is not sufficient
  • b)
    II alone is sufficient while I alone is not sufficient
  • c)
    Either I or II is sufficient
  • d)
    Neither I nor II is sufficient
  • e)
    Both I and II are sufficient
Correct answer is option 'B'. Can you explain this answer?

Shanaya Khanna answered
The colour of fresh grass is 'green' and as given in II, 'green' is called 'brown'. So, the colour of fresh grass is 'brown'.

Chapter doubts & questions for Data Sufficiency - IBPS PO Prelims & Mains Preparation 2024 is part of Bank Exams exam preparation. The chapters have been prepared according to the Bank Exams exam syllabus. The Chapter doubts & questions, notes, tests & MCQs are made for Bank Exams 2024 Exam. Find important definitions, questions, notes, meanings, examples, exercises, MCQs and online tests here.

Chapter doubts & questions of Data Sufficiency - IBPS PO Prelims & Mains Preparation in English & Hindi are available as part of Bank Exams exam. Download more important topics, notes, lectures and mock test series for Bank Exams Exam by signing up for free.

Top Courses Bank Exams

Signup to see your scores go up within 7 days!

Study with 1000+ FREE Docs, Videos & Tests
10M+ students study on EduRev